Quantcast
  • Register
PhysicsOverflow is a next-generation academic platform for physicists and astronomers, including a community peer review system and a postgraduate-level discussion forum analogous to MathOverflow.

Welcome to PhysicsOverflow! PhysicsOverflow is an open platform for community peer review and graduate-level Physics discussion.

Please help promote PhysicsOverflow ads elsewhere if you like it.

News

PO is now at the Physics Department of Bielefeld University!

New printer friendly PO pages!

Migration to Bielefeld University was successful!

Please vote for this year's PhysicsOverflow ads!

Please do help out in categorising submissions. Submit a paper to PhysicsOverflow!

... see more

Tools for paper authors

Submit paper
Claim Paper Authorship

Tools for SE users

Search User
Reclaim SE Account
Request Account Merger
Nativise imported posts
Claim post (deleted users)
Import SE post

Users whose questions have been imported from Physics Stack Exchange, Theoretical Physics Stack Exchange, or any other Stack Exchange site are kindly requested to reclaim their account and not to register as a new user.

Public \(\beta\) tools

Report a bug with a feature
Request a new functionality
404 page design
Send feedback

Attributions

(propose a free ad)

Site Statistics

205 submissions , 163 unreviewed
5,047 questions , 2,200 unanswered
5,345 answers , 22,709 comments
1,470 users with positive rep
816 active unimported users
More ...

  Does quantum mechanics violate the equivalence principle?

+ 10 like - 0 dislike
2205 views

I have a question about equivalence principle in quantum mechanics.

Consider a Schroedinger equation under gravitional field $$\left[ - \frac{1}{2m_I} \nabla^2 + m_g \Phi_{\mathrm{grav}} \right]\psi = i \partial_t \psi \tag{1} $$

where $m_I$ and $m_g$ are the inertia and gravitational masses, respectively. $\hbar=1$ unit is adopted.

To the contrary as the classical mechanics $$ m_I \frac{ d^2 x}{ dt^2} = m_g g \tag{2}$$ we can choose a transformation $x'=x-\frac{1}{2} g t^2$ to "switch off" the gravity. But it seems the transformation will not switch off gravity in quantum mechanics, Eq. (1). Does it mean quantum mechanics break the equivalence principle? (I can think about relativistic Hamiltonian, but it will not resolve the problem, as far as I can see)

This post imported from StackExchange Physics at 2014-03-30 15:22 (UCT), posted by SE-user user26143
asked Aug 1, 2013 in Theoretical Physics by user26143 (405 points) [ no revision ]
There is a nice discussion of this issue in section XIV of arXiv:1205.3365. Summary: "This concludes the section on the weak equivalence principle in quantum mechanics. The result is mixed. We have shown that, very often, it is satisfied in the frame-work of quantum mechanics. But, on the other hand, it seems sometimes to be modified by quantum effects, ... . Maybe this indicates that its application to the more complicated case of quantum vacuum fluctuations of a field is rather suspicious?"

This post imported from StackExchange Physics at 2014-03-30 15:22 (UCT), posted by SE-user Michael Brown
Eq. (871) seems to do the job like "free falling"...although i am not sure about the meaning in (878) yet..

This post imported from StackExchange Physics at 2014-03-30 15:22 (UCT), posted by SE-user user26143
@user26143 : There were errors in the answer, so I delete it.

This post imported from StackExchange Physics at 2014-03-30 15:22 (UCT), posted by SE-user Trimok
I see. Thank you very much for letting me known.

This post imported from StackExchange Physics at 2014-03-30 15:22 (UCT), posted by SE-user user26143

3 Answers

+ 5 like - 0 dislike

The Weak Equivalence Principle, or WEP for short, states that under identical initial conditions, the motion of particles of different masses in a given gravitational field is identical. Or in other words, there are no physical effects that depend on the mass of a point particle in an external gravitational field. This is just the equivalence between the inertial and gravitational mass.

Below I'll present some results that are on either sides of the camp (that WEP is or isn't violated). From what I read, the consensus on this matter is that the equivalence principle in the general sense in violated. However, there are special cases in which the principle holds true for quantum mechanics (one such case in given below).

Let us take the Schrodinger equation for a particle of inertial mass $m_i$ and gravitational mass $m_g$, that is falling toward mass $M$.

$$i\hbar\partial_{t}\psi=-\frac{\hbar^2}{2m_{i}}\nabla^{2}\psi - G\frac{m_{g}M}{r}$$

It is obviously that even for $m_{i}=m_{g}$ the mass does not cancel out of the equations of motions. This fact is even more apparent for $m_{i}=m_{g}=m$ in a uniform gravitational field in the $x$ direction, of acceleration $g$

$$i\hbar\partial_{t}\psi=-\frac{\hbar^2}{2m}\partial_{x}^{2}\psi +mgx\psi$$

whose solution will depend parametrically on $\hbar/m$. At this point we could say that the wave functions, the propagators and probability density distributions violate the WEP. Also, Rabinowitz in the 90's examined the possibility of gravitationally bound atoms, and he found that the mass, $m$, remains in the quantized equations of motions (although the mass cancels out in the classical equations of motions). We would expect $m$ to cancel out when averaging over states with large quantum numbers, but that puts them effectively in the classical continuum.

However, P.C. Davies proposes in this article the following experiment:

Consider a variant of the simple Galileo experiment, where particles of different mass are projected vertically in a uniform gravitational field with a given initial velocity $v$. Classically, it is predicted that the particles will return a time $2v/g$ later, having risen to a height $x_{max}=v^{2}/ 2g$. But quantum particles are able to tunnel into the classically forbidden region above xmax. Moreover, the tunnelling depth depends on the mass. One might therefore expect a small, but highly significant mass-dependant ‘quantum delay’ in the return time. Such a delay would represent a violation of the equivalence principle.

At the end of section $3$ he proves that the expectation value for the turn-around time of a quantum particle is identical, when the measurement is performed far from the classical turning point. In this sense, the WEP holds for a quantum particle.

This result suggest that a uniform gravitational potential—which applies locally to any non-singular gravitational field—has a special property in relation to quantum mechanics, namely that the expectation time for the propagation of a quantum particle in this background is identical to the classical propagation time. This may be taken as an extension of the principle of equivalence into the quantum regime (for a broader discussion of what is entailed by a ‘quantum equivalence principle’). This special property seems to depend on the form of the potential; it does not apply in the case of a sharp potential step, or an exponential potential.

Finally I would like to point out this article, where the authors compute low corrections to the cross-section for the scattering of different quantum particles by an external gravitational field (taken as an external field in linearized gravity). They show that to first order, the cross-sections are spin-dependent. In the second order, they are dependent on energy as well. So, they prove that the equivalence principle is violated in both cases.

This post imported from StackExchange Physics at 2014-03-30 15:22 (UCT), posted by SE-user vnb
answered Dec 3, 2013 by vnb (50 points) [ no revision ]
+1 Thanks for a well written, readily grasped review. I'm still a bit weirded out by all this: I really had no idea of this yet-another-subtlety in bringing GR and QM together.

This post imported from StackExchange Physics at 2014-03-30 15:22 (UCT), posted by SE-user WetSavannaAnimal aka Rod Vance
+ 2 like - 0 dislike

The problem only arises when considering energy eigenstates, which are completely delocalized. One of the tenets of the equivalence principle is that the equivalence between gravitational systems and accelerated frames is only true locally. All that the equivalence principle states is that there is a neighbourhood in any point small enough such that the physical systems are equivalent. But an energy eigenstate will be dependent on the physics outside any such neighbourhood. Because of that, eigenstates being highly delocalized states, do not need to satisfy the equivalence principle in any way

A more detailed explanation would be as follows: The transformation induces a Hamiltonian on the accelerated frame. The equivalence principle says that if the locality is small enough, you can't do a measurement inside the locality that will tell me if I'm in a gravitational potential or in an accelerated frame. But, by definition you can't make perfect momentum or energy measurements inside such locality because your locality has an extent in space, so the position can't be outside of that extent. Hence no possible measurement inside the locality can tell apart extended eigenstates of energy or momentum. If the packets are narrower than the width of the locality, the equivalence principle should hold. If the packets are widers than the locality, the equivalence principle should NOT hold

This post imported from StackExchange Physics at 2014-03-30 15:22 (UCT), posted by SE-user lurscher
answered Dec 8, 2013 by CharlesJQuarra (555 points) [ no revision ]
The time-evolution, Eq. (1), is valid for any state, if i am not mistaken...

This post imported from StackExchange Physics at 2014-03-30 15:22 (UCT), posted by SE-user user26143
Indeed the equation is valid.

This post imported from StackExchange Physics at 2014-03-30 15:22 (UCT), posted by SE-user lurscher
+ 0 like - 4 dislike

If Quantum Mechanics violates the equivalence principle then this would mean that Einstein's theory is wrong. But we know from experiments (Gravity Probe B...etc) that Einstein's theory is not wrong. Thus Quantum Mechanics doesn't violate Einstein's theory so it doesn't violate the equivalence principle.

This post imported from StackExchange Physics at 2014-03-30 15:22 (UCT), posted by SE-user MyFavouritePhysicistIsNewtax
answered Oct 18, 2013 by MyFavouritePhysicistIsNewtax (-40 points) [ no revision ]
This is totally unhelpful until you answer "how wrong does QM say the equivalence principle should be in the actual experiments where it has been checked?" Everything is known only to finite precision, including the equivalence principle. You can't just assume the answer a priori as you have done. You must actually do a calculation.

This post imported from StackExchange Physics at 2014-03-30 15:22 (UCT), posted by SE-user Michael Brown
I should also mention that experiments with ultra-cold neutrons have confirmed the non-trivial mass dependence of gravitational energy levels predicted by quantum mechanics. (The energy is of the order $mg\ell$ where $\ell$ is the de Broglie wavelength $(\hbar^2/m^2 g)^{1/3}$. So the total mass dependence of the energy is $m^{1/3}$.) So the interface of QM and the equivalence principle is indeed a subtle matter. By the way I'm not the downvoter.

This post imported from StackExchange Physics at 2014-03-30 15:22 (UCT), posted by SE-user Michael Brown
@MichaelBrown Wow! WOW!! There's more than enough material in your comments for a highly interesting answer to the uninitiated like me: I had no idea of this yet-another-subtlety in bringing GR and QM together.

This post imported from StackExchange Physics at 2014-03-30 15:22 (UCT), posted by SE-user WetSavannaAnimal aka Rod Vance

Your answer

Please use answers only to (at least partly) answer questions. To comment, discuss, or ask for clarification, leave a comment instead.
To mask links under text, please type your text, highlight it, and click the "link" button. You can then enter your link URL.
Please consult the FAQ for as to how to format your post.
This is the answer box; if you want to write a comment instead, please use the 'add comment' button.
Live preview (may slow down editor)   Preview
Your name to display (optional):
Privacy: Your email address will only be used for sending these notifications.
Anti-spam verification:
If you are a human please identify the position of the character covered by the symbol $\varnothing$ in the following word:
p$\hbar$ysics$\varnothing$verflow
Then drag the red bullet below over the corresponding character of our banner. When you drop it there, the bullet changes to green (on slow internet connections after a few seconds).
Please complete the anti-spam verification




user contributions licensed under cc by-sa 3.0 with attribution required

Your rights
...